Bytelearn - cat image with glassesAI tutor

Welcome to Bytelearn!

Let’s check out your problem:

Let 
h be a continuous function on the closed interval 
[-3,4], where 
h(-3)=-1 and 
h(4)=2.
Which of the following is guaranteed by the Intermediate Value Theorem?
Choose 1 answer:
(A) 
h(c)=-2 for at least one 
c between -1 and 2
(B) 
h(c)=1 for at least one 
c between -3 and 4
(C) 
h(c)=1 for at least one 
c between -1 and 2
(D) 
h(c)=-2 for at least one 
c between -3 and 4

Let h h be a continuous function on the closed interval [3,4] [-3,4] , where h(3)=1 h(-3)=-1 and h(4)=2 h(4)=2 .\newlineWhich of the following is guaranteed by the Intermediate Value Theorem?\newlineChoose 11 answer:\newline(A) h(c)=2 h(c)=-2 for at least one c c between 1-1 and 22\newline(B) h(c)=1 h(c)=1 for at least one c c between 3-3 and 44\newline(C) h(c)=1 h(c)=1 for at least one c c between 1-1 and 22\newline(D) h(c)=2 h(c)=-2 for at least one c c between 3-3 and 44

Full solution

Q. Let h h be a continuous function on the closed interval [3,4] [-3,4] , where h(3)=1 h(-3)=-1 and h(4)=2 h(4)=2 .\newlineWhich of the following is guaranteed by the Intermediate Value Theorem?\newlineChoose 11 answer:\newline(A) h(c)=2 h(c)=-2 for at least one c c between 1-1 and 22\newline(B) h(c)=1 h(c)=1 for at least one c c between 3-3 and 44\newline(C) h(c)=1 h(c)=1 for at least one c c between 1-1 and 22\newline(D) h(c)=2 h(c)=-2 for at least one c c between 3-3 and 44
  1. Apply Intermediate Value Theorem: The Intermediate Value Theorem states that if a function is continuous on a closed interval [a,b][a, b] and NN is any number between f(a)f(a) and f(b)f(b), then there exists at least one cc in the interval [a,b][a, b] such that f(c)=Nf(c) = N. We need to apply this theorem to the function hh and the given values.
  2. Given Function Values: We are given that h(3)=1h(-3) = -1 and h(4)=2h(4) = 2. This means that the function hh takes on the value 1-1 at x=3x = -3 and the value 22 at x=4x = 4.
  3. Analyze Given Options: We need to determine which of the given options is guaranteed by the Intermediate Value Theorem. Let's analyze each option:\newline(A) h(c)=2h(c) = -2 for at least one cc between 1-1 and 22. This option is not guaranteed because the interval [1,2][-1, 2] does not include both endpoints where the function values are given, which are 3-3 and 44.\newline(B) h(c)=1h(c) = 1 for at least one cc between 3-3 and 44. This option is possible because cc11 is between the function values 1-1 and 22, and the interval cc44 includes both endpoints where these function values are given.\newline(C) h(c)=1h(c) = 1 for at least one cc between 1-1 and 22. Similar to option (A), this option is not guaranteed because the interval [1,2][-1, 2] does not include both endpoints where the function values are given.\newline(D) h(c)=2h(c) = -2 for at least one cc between 3-3 and 44. This option is not guaranteed because 1-144 is not between the function values 1-1 and 22.
  4. Correct Answer: Based on the Intermediate Value Theorem, the correct answer is option (B) because it is the only option that satisfies the conditions of the theorem: the value 11 is between the given function values 1-1 and 22, and the interval [3,4][-3, 4] includes the endpoints where these function values occur.

More problems from Intermediate Value Theorem